37. According to the passage, what effect, if any, did the limited number of available R. exoculata have on the outcome of Szuts's experiment?

Answer and Explanation

Your Answer is

Correct Answer is C

Explanation

According to the question stem, I located L70-72 because I found that exoculata's patch contains a lot of visual pigment, so it has no effect on the results of the experiment. Item C fits the question.